The telephone company offers two billing plans for local calls. Plan 1 charges $30 per month for unlimited calls and Plan 2 charges $15 per month plus $0.03 per call.
a. Use an inequality to find the number of monthly calls for which Plan 1 is more economical than Plan 2 .
b. Explain the meaning of the answer to part a.

Answers

Answer 1

The fixed $30 unlimited calls charge for Plan 1 and the initial charge as well as a charge per call of $0.03 calls in Plan 2 gives;

a. The inequality that indicates the number of calls for which Plan 1 is more economical than Plan 2 is presented as follows;

30 > 15 + 0.03•x

Which gives;

x > 500

b. The meaning of the inequality, in part a. is that Plan 1 is more economical than Plan 2 when more than 500 calls are made

What is an inequality?

An inequality makes a comparison between two values that are not equal.

The given parameters are;

The billing plan for Plan 1 is presented as follows;

Charges, C = $30

The charges of the billing plan for Plan 2 is presented as follows;

C = $15 + $0.03×x

Where;

x = The number of calls made per month

a. The inequality that can be used to find the number of monthly calls for which Plan 1 is more economical than Plan 2 is found as follows;

At the start of registration for each plan, we have;

Cost of Plan 1 = $30

Cost of Plan 2 = $15 + $0.03 × 0 = $15

Therefore, Plan 2 is initially more economical than Plan 1

At the point (or number of calls) at which the expenses for both plans are the same, we have;

30 = 15 + 0.03 × x

Which gives;

x = (30 - 15)/0.03 = 500

From the above equation, we have that after 500 calls, the expenses of both plans will be the same

The inequality that gives the number of calls for which Plan 1 is more economical than Plan 2 is therefore;

30 > 15 + 0.03•x

Which gives;

x > 500

b. The meaning of the inequality, x > 500 is that Plan 1 is becomes more economical than Plan 2 when a person makes more than 500 calls in a month

Learn more about inequalities here:

https://brainly.com/question/17136458

#SPJ1


Related Questions

I need to know the answer but I need to know how to get it and work it out

Answers

Let's start by completing the first table.

The questions gives the depth in two times:

- 2 hours after start -> 64 inches of water

- 4 hours after start -> 48 inches of water

From this, we can identifythe units:

- for time, we will use hours

- for depth, we will use inches

The first given values are:

- for time, 2

- for depth, 64

The second given valuesa are:

- for time, 4

- for depth, 48

So, the table is:

Quantity Name | Time | Depth of Pool

Unit | hour | inch

Given Value 1 | 2 | 64

Given Value 2 | 4 | 48

Now, we need to calculate the slope, which can be done using the given points and the equation:

[tex]\begin{gathered} m=\frac{y_2-y_1}{x_2-x_1} \\ m=\frac{48-64}{4-2} \\ m=\frac{-16}{2} \\ m=-8 \end{gathered}[/tex]

So, the slope is -8 inches per hour.

The interceptwe the b in the equation:

[tex]y=mx+b[/tex]

Where y is the depth and x is the time. So, using the first given values and the slope, we can find b:

[tex]\begin{gathered} 64=-8\cdot2+b \\ 64=-16+b \\ b=64+16 \\ b=80 \end{gathered}[/tex]

Thus, the intercept is 80 inches.

So, the formula we have for the pump is:

[tex]y=-8x+80[/tex]

Using it, we can answer questions 1 and 2:

1. After 6 hours means x = 6, so:

[tex]\begin{gathered} y=-8\cdot6+80 \\ y=-48+80 \\ y=32 \end{gathered}[/tex]

So, y = 32, thus, the depth of the water will be 32 inches.

2. The level 24 inches mean y = 24, so:

[tex]\begin{gathered} 24=-8x+80 \\ 8x=80-24 \\ 8x=56 \\ x=\frac{56}{8} \\ x=7 \end{gathered}[/tex]

Thus, x = 7, and the water will be at 24 inches after 7 hours.

Robert Key's group medical insurance coverage costs $6,480 a year. His employer, Covington Arts Center, pays 65% of the cost. What is his monthly paycheck deduction for medical insurance?​

Answers

Answer:

See below

Step-by-step explanation:

Robert pays 35 %   in 12 monthly installments

6480 * .35 / 12 = 189 dollars per month

Printer A prints 36 pages every 1.5 minutes. Printer B prints 114 pages every 3 minutes.
Printer C prints 115 pages every 5 minutes. Which printer prints the fastest?

Answers

Answer:

Printer B prints the fastest

It takes 4 oranges to makes 6 ounces of orange juice. What are the two unit rates?

Answers

Answer: The second ratio in the proportion is set up as ounces over oranges. The units should be in the same place in proportion to the first ratio.

Step-by-step explanation:

Pam needs 2 yards of blue ribbon she has 0.497 yard she buys 0.91 yard more than she buys 0.4 yard of red ribbon how much more blue ribbon does Pam need?

Answers

Total ribbon Pam needs = 2 yards

Total ribbon Pam purchased = 0.497 + 0.91 + 0.40 = 1.807

The amount of ribbon Pam needs is calculated as,

[tex]\begin{gathered} \text{Amount n}eeded\text{ = 2 - 1.807} \\ \text{Amount n}eeded\text{ = }0.193 \end{gathered}[/tex]

Thus the amount of ribbon needed is 0.193 yards .

∠A and \angle B∠B are vertical angles. If m\angle A=(x+12)^{\circ}∠A=(x+12)

and m\angle B=(6x-8)^{\circ}∠B=(6x−8)

, then find the measure of \angle B∠B

Answers

Measure of ∠B = 16°

∠A and ∠B are Vertical angles with ∠A = x+12° and ∠B = 6x-8°.

Vertical angles are angles formed between two lines at the intersection of them and are opposite to each other.

Also measures of Vertical angles are equal.

Now to find the measure of ∠B, we need to find x.

So here,  ∠A = ∠B

⇒ x+12° = 6x-8°

⇒ 6x - x = 12 + 8

⇒ 5x = 20

⇒ x = 4

Hence, ∠B = 6x-8° = 6 x 4 -8 = 16°

So measure of ∠B = 16°

Learn more about angles at https://brainly.com/question/25770607

#SPJ1

Using the bankers rule, find the simple interest on 18,000 pesos at 17.25% from Feb.4 to Apr 21 of the same leap year.

Answers

From the question

By the bunkers rule

[tex]I=\text{PRT}[/tex]

P = 18,000

R = 17.25% = 0.1725

For the time

Since the year is a leap year then

February has 29 days

Hence

From Feb.4 to Apr 21 implies

The number of days left in Feb. = 25

Number of days in March = 31

Number of days for April = 21

Therefore, Total number of days = 25 + 31 + 21 = 77

By Bankers rule, total days for the year will be 360

Hence the interest is

[tex]\begin{gathered} I=18000\times0.1725\times\frac{77}{360} \\ I=18000\times0.1725\times0.2139 \\ I=664.16 \end{gathered}[/tex]

Therefore, the interest is 664.16

ASAP!!!!

PLEASE HELP!!!

When a whole number is multiplied by 10 to the power of 2, how does the placement of the decimal point change?


a) It moves two places to the right.

b) It moves ten places to the right.

c) It moves two places to the left.

d) It moves ten places to the left.

Answers

Answer:  A) moves two places to the right

Example:

[tex]2.567 * 10^2 = 2.567 * 100 = 256.7[/tex]

The positive exponent over the 10 tells us how many places to move to the right. If the exponent was negative, then we'd move to the left. This only works when the base is 10.

From the diagram below, what is the measure of the angle of depression from point A

Answers

The angle of depression is the angle between the horizontal line and the observation of the object from the horizontal line.

From point A, point B is observed with an angle of 45 down from the top of the tree to the ground.

Thus, the angle of depression is 45°

In the circle below, the central angle is measured in radians.When rounded to the nearest .01 of a centimeter,the length of arc Sis?? cm?

Answers

Length of the given arc

[tex]\begin{gathered} l=r\theta \\ =2\times\frac{\pi}{4} \\ =\frac{\pi}{2} \\ =\frac{22}{14} \\ =1.571 \\ =1.57 \end{gathered}[/tex]

So the length of the given arc is 1.57 cm

Write a sequence of transformations that takes A to A'.

Answers

A rotation of 270 degrees about the origin would transform the figure as follows:

Which does not maps the A to A'.

Now, one sequence of transformations that take A to A' is:

1)First, a reflection over the x-axis would transform the figure as follows:

2) We translate the figure 3 units up and 4 units to the right:

Answer: Reflection over the x-axis, translation 3 units up and 4 units to the right.

A brick has dimensions of110. cm x 655 cm x 1330 cm.What is the volume of the brick incubic meters?[?] m3Recall that 1 cm =10-2 mGive your rounded answer to the correct number ofsignificant figures.Volume, cubic metersEnter

Answers

The dimension of the brick is given 110 cm x 655 cm x 1330 cm.

We know that 1 cm = 10^-2 m.

So, the dimensions of brick in meters will be 1.10 m x 6.55 m x 13.30 m.

The volume of the brick is calculated below:

[tex]\begin{gathered} V=1.10\times6.55\times13.30 \\ =95.8265m^3 \end{gathered}[/tex]

Thus, the volume of the brick in cubic meters is 95.8265 m^3.

please helpppppppppppppp

Answers

Answer:

Step-by-step explanation:

Substitute a and b with 1/4 and 6

5b/ 32a^2

5(6)/ 32(1/4)^2

30/32(1.96)

30/62.72

0.5102 Answer

Part A: On the imagePart B: What is the answer of k?

Answers

The effect on the graph replacing f(x) by f(x)+k is a vertical shift k units.

According to the graph the resulting function is 2 units higher from the original one. This means k has a value of 2.

72b384 is a number in which one of its digits is 'b'. if the number is a multiple of 9, what is the numerical value of 'b'?

Answers

Answer:

3

Step-by-step explanation:

Hello!

A simple way to find if the number is a multiple of 9 is if the sum of the digits in the number is a multiple of 9, then the number itself is a multiple of 9.

So that means: 7 + 2 + b + 3 + 8 + 4 is a multiple of 9

If we add it up we should get: 24 + b

The multiples of 9 include 9,18,27,36,45,54....., and the closest multiple of 9 to 24 is 27. Therefore, the missing digit should be 3, as 27 - 24 is 3.

The next closest numbers would be 18 and 36, but you would need to add a negative number to get 18, and a two-digit number to get 36.

Solve the equation for all values of x.
|5x + 8| 8 = 3x

Answers

the answer is x equals 0 and -2

Use 2 methods to predict the output for an input of 200.

Answers

Given a table represents a relation between x and y

As shown, the change of (x) and (y) vary with a constant rate

So, the table represents a linear function

The general equation will be: y = m * x + b

where (m) is the slope, (b) is the value of y when (x=0)

So, from the table, when x = 0 , y = 25

The slope will be as follows:

[tex]m=\frac{30-25}{1-0}=\frac{5}{1}=5[/tex]

So, the equation will be:

[tex]y=5x+25[/tex]

When the input = 200, x = 200

So, substitute with (x) to find the output (y)

[tex]y=5\cdot200+25=1000+25=1025[/tex]

So, the answer will be:

For an input 200, the output is 1025

===================================================

Another method to solve using the arithmetic sequence.

As shown in the table consider (x) represents the number of terms

So, (y) will be an arithmetic sequence

The first term = a = 30

The common difference = d = 35 - 30 = 5

the general rule of the arithmetic sequence is:

[tex]y_{}=a+d(x-1)[/tex]

substitute with (a) and (d)

[tex]y=30+5(x-1)[/tex]

When x = 200

[tex]y=30+5(200-1)=30+5\cdot199=1025[/tex]

So, for an input 200, the output = 1025

Carefully follow the steps to find the solution to the three equation system.

a. Use equations 2 and 3 and eliminate the by multiplication and addition, creating a new equation with only two variables.
b. Use equations 1 and 2 and eliminate the by multiplication and addition, creating a second equation with only two variables.
c. Use the two new equations, and eliminate the -variable by multiplication and addition, finding the value for the -variable.
d. Substitute -value in the second new equation and find the -value.
e. Substitute the and values into original equation 2 to find the -value

(-2,-5,-3)
(1,4,2)
(-4,1,-2)
(1,2,4)
(2,5,3)

Answers

Answer:

The answer is (1,4,2)

Solve each system of equations please show your work! 3x+y-2z=22 x+5y+z=4 x=-3z

Answers

The solution of the system of equations are;

⇒ x = - 6 , y = 44 and z = 2

What is substitution method?

To find the value of any one of the variables from one equation in terms of the other variable is called the substitution method.

Given that;

The system of the equations are,

3x + y - 2z = 22    .... (i)

x + 5y + z = 4       .... (ii)

x = - 3z                .... (iii)

Now,

Solve the equations as;

Substitute x = - 3z in both equations, we get;

⇒ 3x + y - 2z = 22

⇒ 3 × -3z + y - 2z = 22

⇒ - 9z + y - 2z = 22

⇒ - 11z + y = 22   .... (iii)

And, x + 5y + z = 4

- 3z + 5y + z = 4

5y - 2z = 4     ... (iv)

Solve equation (iii) and (iv) we get;

⇒ y = 44 and z = 2

So, x = - 3z    

⇒ x = - 3 × 2

⇒ x = - 6

Thus, The solution of the system of equations are;

⇒ x = - 6 , y = 44 and z = 2

Learn more about the system of equations visit:

https://brainly.com/question/26111659

#SPJ1

(2 3/5) Multiply (-2 2/3)

Answers

[tex] \frac{13}{5} \times - \frac{8}{3} \\ = - \frac{104}{15} [/tex]

ATTACHED IS THE SOLUTION

Answer:

(-104/15)

Step-by-step explanation:

    3              2

2 ------ × -2 -------

    5              3

5 × 2 = 10

10 + 3 = 13

3 × -2 = -6

-6 + -2 = -8

 13        -8            104

------- × ------- = (-) -------

  5          3             15

I hope this helps!

Match the terms (picture)

Answers

Answer:

Match that i have the step by step explanations

Step-by-step explanation:

A, F, C, D, E, F, B

9. You are working to determine how much paint you will need to paint your perfectly square room. The
length of the wall is 4y+8. The height of each wall in your room is 4y. What is the area of the first wall

Answers

The area of the wall whose length is 4y+8 and the height is 4y is 16y^2 + 32y.

How area is calculated?

Area is a phrase used to describe how much room a 2D form or surface occupies. Area is measured in square units, such as cm2 or m2.

The area of a form is computed by dividing its length by its breadth.

It is given that the length of the wall is 4y+8 and height of the wall is 4y.

From this given information, we can conclude that the wall is rectangle.

Formula to calculate the area of rectangle is:

Area of rectangle = length x breadth

Putting the value given in the question:

Area of the wall = (4y+8)(4y)

Area of the wall = 16y^2 + 32y

Therefore, the area of the wall is 16y^2 + 32y

To know more about area, go to link

https://brainly.com/question/25292087

#SPJ9

Distribution Whole Number Multiple Choice

Answers

Distribution of

a(b + c) = ab + ac

Then

36 + 12 is equivalent to

Find composed numbers

36= 3•12

12 = 3•4

THEN ANSWER IS

3• (12 + 4)

what is the lcm of the rational algebraic equation 6/x+x-3/4=2

Answers

The required answer would be (24 -3x - 4x²) / 4x = 0 which is the lcm of the rational algebraic equation 6/x+x-3/4=2.

What is the equation?

The equation is defined as mathematical statements that have a minimum of two terms containing variables or numbers that are equal.

The given equation below as:

⇒ 6/x + x - 3/4 = 2

We have to determine the lcm of the rational algebraic equation

⇒ 6/x + x - 3/4 = 2

Rearrange the term of 2 in the equation,

⇒ 6/x + x - 3/4 - 2 = 0

Take LCM in the above equation,

⇒ [tex]\dfrac{6\times4+x\times4x-3\times x -2 \times 4x}{4\times x}[/tex]

⇒ (24 + 4x² -3x - 8x²) / 4x = 0

Combine the likewise terms in the numerator,

⇒ (24 -3x - 4x²) / 4x = 0

Therefore, the required answer would be (24 -3x - 4x²) / 4x = 0.

Learn more about the equation here:

brainly.com/question/10413253

#SPJ1

Graph 5x + y = 3x + 2(a) Rewrite the equation in slope intercept form. Show your work.(b) Graph the line. If you aren't able to draw on the graph using the technology you have available,then just describe it in words - tell where it is located

Answers

Given the Linear Equation:

[tex]5x+y=3x+2[/tex]

(a) You need to solve for "y", in order to rewrite it in Slope-Intercept Form, because that form is:

[tex]y=mx+b[/tex]

Where "m" is the slope of the line and "b" is the y-intercept.

Then, you get:

[tex]\begin{gathered} y=-5x+3x+2 \\ y=-2x+2 \end{gathered}[/tex]

Notice that:

[tex]\begin{gathered} m=-2 \\ b=2 \end{gathered}[/tex]

(b) You can graph the line by finding the x-intercept and the y-intercept. You already know that the y-intercept is:

[tex]b=2[/tex]

Then, in order to find the x-intercept, you need to substitute this value of "y" into the equation and then solve for "x":

[tex]y=0[/tex]

Because the value of "y" is zero when the line intersects the x-axis.

Therefore, you get:

[tex]\begin{gathered} y=-2x+2 \\ 0=-2x+2 \\ -2=-2x \\ \\ \frac{-2}{-2}=x \\ \\ x=1 \end{gathered}[/tex]

Now you know that the line passes through these points:

[tex](1,0),(0,2)[/tex]

Hence, you can graph it.

Therefore, the answers are:

(a) Equation in Slope-Intercept Form:

[tex]y=-2x+2[/tex]

(b) Graph:

Linh buys $11.23 worth of groceries and pays for them with a $20 bill.List her change using the smallest number of coins and bills?

Answers

Given: Linh buys $11.23 worth of groceries and pays for them with a $20 bill.

So, the change will be = 20 - 11.23 = $8.77

9. Assessment Practice Diego wants to
find 8+ 9. 2.NSO.2.1
Which doubles fact can help him?
A 5+5=10
B6+6=12
C7+7= 14
D 8+8=16

Answers

They can use counters or their fingers, but they can also use doubles facts. If they know 5 + 5 = 10, they know that the sum of 5 + 6 will be one more.

What is meant by addition?

One of the four fundamental mathematical operations, along with addition, subtraction, multiplication, and division, is addition.

The sum of 5 + 6.

They can utilize counters or their fingers, but they can even utilize doubles facts. If they understand 5 + 5 = 10, they know that the sum of 5 + 6 will be one more.

Therefore, 5 + 6 = 11.

Estimating other doubles-plus-one facts together, such as 3 + 4, 7 + 8, and 9 + 8.

Repeat the doubles facts so they can use them to solve equations.

They can also use doubles-minus-one facts to enable them solve equations.

Write down 9 + 10 and have your children find the sum.

If they know that 10 + 10 = 20, they know that the sum of 9 + 10 will be one less. Therefore, 9 + 10 = 19.

Practice solving different number sentences together.

Therefore, they can utilize counters or their fingers, but they can even utilize doubles facts. If they understand 5 + 5 = 10, they know that the sum of 5 + 6 will be one more.

Therefore, the correct answer is option A 5 + 5 = 10.

The complete question is:

Which doubles fact helps you solve 5 + 6 = 11? Circle the number sentence.

A 5+5=10

B6+6=12

C7+7= 14

D 8+8=16

To learn more about addition refer to;

https://brainly.com/question/1380218

#SPJ13

Please help I did help

Answers

The scientific notation of the expression is as follows:

(7 × 10⁻⁶) (7 × 10⁻⁶) = 4.9 × 10⁻¹¹(3.76 × 10⁵) + (7.44 × 10⁵) = 1.12 × 10⁶

How to solve expression in scientific notation?

The proper format for scientific notation is a x 10ᵇ where a is a number or decimal number such that the absolute value of a is greater than or equal to one and less than ten or, 1 ≤ |a| < 10.

Scientific notation is a special way of writing numbers.

The expression can be express in scientific notation as follows:

Therefore,

(7 × 10⁻⁶) (7 × 10⁻⁶)

Hence, we have to multiply the integers and add the exponents as follows:

(7 × 7) × (10⁻⁶. 10⁻⁶)

49 × 10⁻⁶⁻⁶

49 × 10⁻¹²

4.9 × 10⁻¹¹

Hence, the second can be express in scientific notation as follows:

(3.76 × 10⁵) + (7.44 × 10⁵)

(3.76 + 7.44) × 10⁵

11.2 × 10⁵

1.12 × 10⁵⁺¹

1.12 × 10⁶

learn more on scientific notation here: https://brainly.com/question/15372705

#SPJ1

Ben is a coal miner and spends most of his workday in an underground mine. Which is the most likely depth he is at while doing his job?

Answers

Answer:-900 feet. I think this is correct so don't use it unless you think its right

Step-by-step explanation:

Amanda and Jeremiah are standing on the same side of a large lake. They are separated by a horizontal distance of 4000 ft. Both Amanda and Jeremiah can see a helicopter that is directly above the horizontal distance between them. Amanda’s angle of elevation to see the helicopter is 60 and Jeremiah’s angle of elevation to see it is 30. 1.What is the distance between Amanda and the helicopter?

Answers

Explanation: First of all we need to know if our triangle is a right triangle (has one 90° inner angle) to be able to use the law of sines to calculate the distance between Amanda and the helicopter.

Step 1: Once we know that the sum of all inner angles of a triangle is equal to 180° we can calculate the unknown angle as follows

As we can see above the missing inner angle is 90° which means it is a right triangle.

Step 2: Now we can use the law of sines as follows

once sin (90°) = 1 and sin (30°) = 0.5, let's calculate "x" below

[tex]\begin{gathered} \frac{x}{sin(30)}=\frac{4000}{sin90} \\ x=\frac{4000*0.5}{1} \\ x=2000ft \end{gathered}[/tex]

Final answer: As we can see above, the final answer is 2000 ft

Other Questions
an athlete consumes large amounts of meat in an effort to build extra muscle tissue. this practice does not work because: Since wY, XZ, and AB intersect at 0, WOA ~ BOY andXOB by the vertical angles theorem. It is given thatAOZ. By the transitive property, BOY XOB.AOZWOAWhat is the conclusion reached by this proof? Which word form correctly completes the following sentence?Leo was surprised when the saleslady told him the price of the car was not _____.A.negotiatedB.negotiableC.negotiationD.negotiating you are studying a chromosome in a new animal species. the presence of what element would make you most confident that a region is facultative heterochromatin and not constitutive heterochromatin? What is the figurative meaning of the moon in the passage? A. sleep B. soothing feelings C. light D. passing of time For which of the following situations is the taxpayer not required to file Schedule H - Household Employment Taxes? asquarematrixiscalledapermutationmatrixifitcon- tains a 1 exactly once in each row and in each column, with all other entries being 0. examples are in and are permutation matrices invertible? if so, is the inverse a permutation matrix as well? How many positive real zeroes does f(x) = x - 4x + 7x + 3x - 5 have? Daniel walks at a speed of 4 miles per hour. Daniel leaves his house at 1:00 p.m. and walks toward a playground exactly 2 miles away. Daniel's mother plans to walk to the playground too. She walks at a speed of 6 miles per hour. At what time should Daniel's mother leave the house to arrive at the playground at the same time as Daniel? Use the table to find the products od the two polynomials. Write your answer in descending order l. Gabriel bought 12 donuts and fritters from thebakery. Donuts cost $1.00 and fritters cost$2.00. Gabriel spent a total of $17.00. Howmany donuts (d) and how many fritters (f) didhe buy?d+f=121d + 2f = 17[?] donuts [?] fritters How do u solve this? satisfy this equation x + |y| = y + |x| IF -10 x 10 and -10 y 10 cardiorespiratory fitness is important because it multiple choice question. stretches the small muscles of the body. rests the heart and lungs while exercises are performed. allows the blood to carry oxygen and nutrients to the body. blocks the body's glands from producing hormones. Simplify The answer please Marsha used a graphing calculator to graph an expense and a revenue function for her familys printing business. The graph looked like the one below. analyze the extent to which our criminal justice system is effective.CAN SOMEONE EXPLAIN THEIR UNDERSTANDING TO THIS PROMPT QUESTION Which planet is not a gas planet?A. UranusB. JupiterC. VenusD. Neptune What change must made to the coefficients to balance this equation?C3H8 + 5 O2 4 CO2 + 3 H2O A cone has a volume of 3014.4 cubic inches and a radius of 12 inches.what is the height?